Most Strongly Supported Logical Reasoning Questions

LSAT Blog Most Strongly Supported Logical Reasoning
Many prep companies make the mistake of failing to distinguish between Most Strongly Supported and Must Be True.


They place "Most Supported" / "Most Strongly Supported" Logical Reasoning questions under the category of "Must Be True" questions. This is incorrect.


It's understandable.

After all, the answer to a Most Strongly Supported question is sometimes something that Must Be True. After all, something that Must Be True based upon the stimulus can certainly be said to be the Most Strongly Supported of the answer choices. It's 100% supported by the stimulus, and you can't support something more than 100%.

For this reason, you can sometimes treat Most Strongly Supported questions like Must Be True / Inference questions.

However, keep in mind that the test-writers have some wiggle-room with Most Supported questions. The question is not asking for what Must Be True, so our task is not always to find something that Must Be True.

Instead, we must look for what's Most Supported among the choices, and the correct answer choice will not necessarily be supported 100% of the way. While it's certainly easier when the correct answer happens to be something that Must Be True, the correct answer choice does not necessarily have to be true. If you find yourself staring at 5 choices, and none of them appears to be a Must, look for the answer choice that's Most Likely To Be True.

To summarize:
It's incorrect to treat Most Strongly Supported questions as Must Be True questions because the correct answer doesn't necessarily need to be true. It's just the answer choice that's most likely to be true.

Examples:
I've picked two examples from The Next 10 Actual Official LSAT PrepTests, and an example from a recent LSAT PrepTest, to illustrate the difference.

First, a Most Strongly Supported question where the correct answer Must Be True:

PrepTest 36 (December 2001), Section 1, Question 16 (page 258 in Next 10)

The evidence gives examples of countries with viable economies that have populations of fewer than 7 million - everything listed aside from Switzerland and Austria, which leaves Israel, Ireland, Denmark, and Finland.

Therefore, it must be true that economic viability doesn't require having a population of at least 7 million.

***
Now, a Most Strongly Supported question where the correct answer is Most Likely to Be True but is not necessarily something that Must Be True:

PrepTest 33 (December 2000), Section 3, Question 13 (page 171 in Next 10)

To say that the credited response Must Be True would be to commit the post hoc fallacy (a type of correlation/causation fallacy). Yes, the argument in the stimulus states that once ramps, etc., are installed, wheelchair users come there. However, this is only a correlation.

The sudden arrival of wheelchair users could simply be a coincidence, or perhaps there's something else going on - some third variable at play.

For example, maybe the wheelchair users didn't go to those areas in the past for other reasons. Maybe the non-accommodation-providing business areas were managed by jerks, and their jerkitude was the real reason the wheelchair users didn't frequent the area. Maybe the new business area managers happen to be nice people and the wheelchair users like to hang out with them.

However, out of the 5 choices, it is Most Likely To Be True that the wheelchair users didn't come there in the past due to the lack of proper accommodations.

***

I'm also explaining a Most Strongly Supported question in a recent exam where the answer did not necessarily have to be true.

I'm posting this explanation in the comments to avoid corrupting this recent LSAT PrepTest for those of you who intend to take this as a full-length timed exam. If you like, take a quick peek at the bolded portion of the comment to see if it's from an exam you intend to save for later.

***
You can find more information about the difference between Must Be True and Most Strongly Supported on pages 20-22 of LSAC's SuperPrep.

Photo by ms_cwang / CC BY-NC-SA 2.0



11 comments:

  1. Now, an example from a recent exam where the answer to a Most Strongly Supported question is something that is Most Likely to Be True but does not have to be true:

    June 2007, Section 2, Question 18 - Most researchers / climate change
    (page 13 of the free June 2007 LSAT PDF, numbered as page 11 in the upper-right corner)

    Given incentives, etc., the credited response is Most Likely To Be True. However, it doesn't have to be true. In fact, with regard to this stimulus, another LSAT question could have easily pointed out a vulnerability (flaw) - something the author failed to consider. The same is true of the above example with the wheelchair users, by the way.

    Perhaps scientists aren't motivated by recognition. Maybe they just want to be left alone to run their experiments and studies without any paparazzi hassling them. In that case, they would have a *disincentive* to go against the norm and make waves. They can still show that other, less sexy, hypotheses are incorrect (this is, after all, the foundation of modern science according to the stimulus' evidence) without discrediting the global warming hypothesis in particular, which might bring unwanted attention.

    ReplyDelete
  2. Still trying to figure this out so forgive my ignorance.

    In PT 33, III, 13 it looks like there is a sufficient/necessary situation in the stimulus:

    Ramps etc. installed —> Wheelchair folks come

    And the contrapositive would be:

    Wheelchair folks don't come —> No ramps etc. installed

    Does this not make it a Must Be True question? Or perhaps the wording does not necessitate that. What do you think?

    ReplyDelete
  3. This is a great question, Evan, and I'm glad you asked it. It's important to fully engage with everything you come across.

    This relationship in the stimulus is temporal (time-related). Just because ramp installations, etc., occur before wheelchair users' arrivals, this doesn't mean the ramp installations, etc., themselves caused the arrival of the wheelchair users. (That would be the correlation/causation post hoc fallacy.)

    The ramps, etc., installations could simply be a signal/effect of something else (like nice/mean business area managers).

    To directly answer your first (implied) question: Yes, this is a sufficient/necessary situation. Your diagramming is accurate.

    However, the contrapositive : No WC ---> No ramps, etc. is not what the credited response (correct answer) says.

    The correct answer contains information that is not 100% supported by the stimulus. We don't know for certain that it is the lack of proper accommodations itself preventing the WC folks from coming to the business area.

    Although it is quite likely that it is, it is not necessarily the case.

    ReplyDelete
    Replies
    1. Super helpful distinction between claimed causal effect and a guaranteed trigger-result relationship, if I'm reading this correctly. Thanks, Evan & Steve for this exchange!

      Delete
  4. Hey steve, If many can encompass all, can some encompass none(no one)? an example of this would be in test 29 ques. #18 the answer is E) "Some persons ....do not live in the suburbs" but the stimulus clearly states that "no one on the planning committee lives in the suburbs.."

    ReplyDelete
  5. Hi Steve, been following your blog for some time. I think another good example is PT 39, Sec. 4, Q. 16? The one regarding Cezanne as an early modernist? Not sure if this fits the bill. Thanks.

    ReplyDelete
  6. WOW, if what you say is correct then you have explained what I have been struggling with the entire day. Could you please take a look at DEC 2001 Section 1 #4 about antidepressants? I don't think the answer is a Must Be True in that the people taking the drugs could be only taking those that do NOT cause weight gain (since only MOST of the drugs cause weight gain). PLEASE let me know if you agree. Thanks!

    ReplyDelete
  7. http://www.wired.com/science/discoveries/magazine/16-07/pb_theory/#/

    Take a look, correlation is all that matters in our Google world!

    ReplyDelete
  8. Ok, having now gone through all the MBT and Most Strongly Supported questions from PTs 1-20...DUH. The MSS questions are almost entirely different questions to me. The normally strict standard I apply to MBT does not hold for MSS, so I have in as many as 3 or 4 cases eliminated all of the answers and been unhappy with having to choose in the first place. These questions require turning off the strict rules--in that way they remind me of some of the reading comprehension questions that ask, for instance, which of the following would the author most likely support, i.e. the other MSS questions on the test, and in general one of the more uneasy questions available in my view.

    ReplyDelete
  9. Steve,

    I am having a lot of trouble with PT 38 S4, #25. On your study guide you have it listed as a MBT question, but is it a MBT (Except) question since it read:
    The claims made above are compatible with each of the following EXCEPT:

    I am so confused by this question because it seems that the correct answer (D) is the only one that is compatible instead of being the only one that is not compatible.

    ReplyDelete
  10. I really enjoyed your article on most strongly supported questions. I would like to bring something to your attention. I discovered two different types of MSS questions:

    1. “The statements above, if true, most strongly support which one of the following?”
    2. “Which of the following, if true, most strongly supports the statement above?”

    I think that the first MSS is a subset of Must Be True.
    The second MSS is a strengthening question.

    If a student assumes that any stem that mentions "most strongly supported" is a subset of MBT, then they might miss that question.

    ReplyDelete